LSAT and Law School Admissions Forum

Get expert LSAT preparation and law school admissions advice from PowerScore Test Preparation.

 Administrator
PowerScore Staff
  • PowerScore Staff
  • Posts: 8917
  • Joined: Feb 02, 2011
|
#26239
Complete Question Explanation
(The complete setup for this game can be found here: lsat/viewtopic.php?t=10888)

The correct answer choice is (D)

If exactly two employees receive $5k bonuses, then either L or M (but not both) must receive a $5k bonus (the other employee who receives a $5k bonus is X). But, since neither L nor M can receive a $1k bonus (last rule), it follows that one of them must receive a $3k bonus:
June15_game_1_#5_diagram_1.png
Last, we need to ensure that K and P receive lower bonuses than either L or M. So, K and P must receive $1k bonuses:
June15_game_1_#5_diagram_2.png
This inference agrees with answer choice (D), which is correct.

Answer choice (A) is incorrect, because L could also receive a $5k bonus.

Answer choice (B) is incorrect, because M could also receive a $5k bonus.

Note that just like K and P are functionally identical (see discussion in Question #3), so are L and M: both are employees in the same department, and both are Highly Effective. Thus, if answer choice (A) were true, then answer choice (B) would have to be true as well (and vice versa). Given the unique nature of the correct answer choice, we can safely eliminate both (A) and (B).

Answer choice (C) is incorrect, because M need not receive a $5k bonus.

Answer choice (D) is the correct answer choice. See discussion above.

Answer choice (E) must be false, because P receives a $1k bonus, not a $3k bonus.
You do not have the required permissions to view the files attached to this post.

Get the most out of your LSAT Prep Plus subscription.

Analyze and track your performance with our Testing and Analytics Package.